You are on page 1of 36
rey LE CLAT2.0, READY: LEGALEDGE TEST SERIES " art of the most Comprehensive Classroom Training, Prep Content & Test Series across the Nation. From the producers of A.l.R. 2, 3 and 5 in CLAT 2019, MOCK COMMON LAW ADMISSION TEST 2020 MOCK CLAT #19 Candidate Name Batch Contact No. Date of Exam LEGALEDGE INSTRUCTIONS TO CANDIDATES Duration, 120 Minutes Max. Marks 150 Centre Name No clarification on the question paper can be sought. Answer the questions as they are There are 150 multiple choice objective type questions. Each question carries ONE mark. Total marks are 150. There is a negative marking of 0.25 marks for every incorrect answer. Candidates have to indicate the correct answer by darkening one of the four responses provided, with a BALL PEN (BLUE OR BLACK) in the OMR Answer Sheet Example: For the question, "Where is the Taj Mahal located?", the correct answer is (b). The candidate has to darken the corresponding circle as indicated below (a) Kolkata (b) Agra (0) Bhopal (d) Delhi Right Method Wrong Methods ®@©O0 ®®BOOQ @™BOO GH®OO 6. Answering the questions by any method other than the method indicated above shall be considered incorrect and no marks will be awarded for the same. More than one response to a question shall be counted as wrong. The candidate shall not write anything on the OMR Answer Sheet other than the details required and in the spaces provided for. 9. After the Test is over, the candidate has to return the OMR Answer Sheet to the invigilator. The candidate should take the Test Paper along with them. 10. The use of any unfair means by any candidate shall result in the cancellation of his/her candidature. 11. Impersonation is an offence and the candidate, apart from disqualification, may have to face criminal prosecution, 12. Electronic gadgets like mobile phones, pagers or calculators are strictly not permitted inside the Test Centre/ttall 13. The candidates shall not leave the hall before the Test is over. mock cLaTits 4 LEGALEDGE Directions (@.1 - Q.29): Read the passages carefully and answer the questions (Q.1-Q.5): The Wright brothers did not have to look far for ideas when building their airplane, they studied birds. The act of copying from nature to address a design problem is not new, but over the last decade the practice has moved from obscure scientific journals to the mainstream. The term ‘biomimicry’, popularized by American natural-sciences writer Janine Benyus in the late 1990s, refers to innovations that take their inspiration from flora and fauna. Biomimicry advocates argue that with 3.8 billion years of research and development, evolution has already solved many of the challenges humans now encounter. Although we often see nature as something we mine for resources, biomimicry views nature as a mentor. From all around the globe, there are countless instances where natural sources have served as inspiration for inventions that promise to transform every sector of society, One such instance occurred in 1941 when ‘Swiss engineer, George de Mestral was out hunting with his dog one day when he noticed sticky burrs, with their hundreds tiny hooks, had attached themselves to his pants and his dog's fur. These were his inspiration for Velcro. 1, The airplane was inspired by (@) animals (b) plants (0) birds (d) flies 2. Biomimicry refers to designs that (a) are inspired by natural things (b) transformed society (0) are based on scientific engineering _—_(d) arise out of man’s creativity 3. Biomimicry views the natural world as a (a) mine for resources (b) mine field of ideas (c) mentor (d) source of perspiration 4, What has helped solve many of the challenges encountered by man? (@) Biomimicry (b) Evolution (0) Innovation (4) Invention 5. The two instances of biomimicry mentioned in the passage are () flora and fauna (b) birds and burrs (0) copying and innovating (4) airplane and Velcro (Q.6 — Q.8): Homo economicus, or economic human, denotes the idea of human beings as rational, narrowly selfinterested agents who, given total information about opportunities and possible constraints, seek to obtain the highest possible well-being for themselves at the least possible cost. In the late 19th century, a host of economists built mathematical models based on conceiving of real humans as Homo economicus. Exponents of Homo economicus tend to acknowledge that total information is not possible in the real world; thus, breakdown in models based on the concept are due to imperfect information held by the selfinterested economic actors. Amartya Sen has pointed out that Homo economicus ignores that people can and do ‘commit to courses of action out of morality, cultural expectations, and so forth. Veblen and Keynes allege that Homo economicus assumes far too great an understanding of macroeconomics on the part of humans. ‘Tversky put forth that investors are not rational: they are unconcerned by small chances of large losses, but quite risk-averse regarding small losses. Bruno Frey points out that humans are often intrinsically motivated, and that such motivation explains heroism, craftsmanship, and other drives that do not fit neatly into the model of a narrowly focused gain- seeker. Critics of the psychoanalytic tradition point out, somewhat obviously, that humans are frequently conflicted, lazy, and inconsistent mock cLaTits 4 LEGALEDGE 6. Consider each of the answer choices separately and indicate all that apply Which of the following phenomena would exemplify Bruno Frey's critique of Homo economicus? (a) Awoodworker spends months on the delicate inlay of a door, knowing that his many hours of hard work will inevitably result in a higher price when he comes to sell the piece. (b) A television journalist often travels to dangerous countries all over the world because he is contractually obligated to do so, and his ratings are higher the more dangerous his exploits appear to be. (0) An economist dedicates her career to illustrating a fundamental flaw in a particular theory, though she knows there will be no tangible reward for her efforts. (a) None of the above 7. Which of the following best describes the main idea of the passage? (a) Homo economicus is a useful, if theoretical, actor to use in the formation of mathematical models. (b) Homo economicus is a fundamentally flawed and thus theoretically useless construction, for a host of reasons. () Homo economicus is often criticized by those who don''t fully understand its function in economic theory. (d) Homo economicus is a problematic construction, because it simplifies human motivations and is overly optimistic about human understanding 8. Consider each of the answer choices separately and indicate all that apply. Which of the following is a complaint leveled against the theory of Homo economicus posited by certain economists in the late 19th century? (a) It assumes that the average person knows a lot more about the general workings of the economy than he or she actually does. (b) It assumes that humans experience a proportional and linear emotional response to all risks and rewards, (o) It assumes that the primary impetus behind human decision-making is not predicated on ethics or cultural mores. (d) Both (a) and (c) (Q.9 - Q.11): Quantum mechanics is a relatively new field of physics that was developed in the early 1900's. Although we classically think of a particle as a fixed object, quantum mechanics describes particles as waves using properties such as position and energy. The quantum mechanical wave describes the probability of a particle to attain certain values of these properties, and measuring the particle samples from this probability. Take, for example, the analogy of rolling a six-sided die. For each roll there is a one- in-six chance that any single number will result. After rolling, however, only one single number will be observed. If the die is rolled enough times, one can deduce that the die has six sides and that each side is equally likely. However, one can never be completely sure, because rolling dice is probabilistic in nature. Quantum mechanics states that the same is true of the position (and other properties) of a particle. A particle trapped in a closed box has some finite probability of being at any location within the box. Open the box once and you'll find the particle at only one location. Open the box enough times and you'll see all the particle locations and the frequency at which they are achieved. From this, one can deduce the original properties of the quantum mechanical wave, just as one could deduce the properties of the die The counterintuitive properties of quantum mechanics, that the attributes of a particle cannot be known in advance of measurement, initially provoked many strong philosophical debates and interpretations regarding the field. In fact, Einstein was deeply troubled by the idea of nature being probabilistic and commented famously that, “God does not play dice with the universe.” Over the last 70 years, however, irrefutable evidence has abounded that verifies the truth of the theory of quantum mechanics. 9. Which of the following best expresses the main idea of the passage? (a) Particles are not a fixed object but rather waves. (b) Controversial theories are often found to be correct. (c) Quantum mechanics correctly postulates the probabilistic nature of particles. (d) Many questions still exist about the nature of particles and quantum mechanics. mock cLaTits 4 LEGALEDGE 10. Based on the information in the passage, which of the following would best explain Einstein's motivation for stating that “God does not play dice with the universe”? (a) Einstein did not believe that particles should be governed by probability as in a game of dice. (b) Einstein believed that God should control the fate of the universe (c) Einstein was opposed to the theory of quantum mechanics on the grounds that it violated causality. (d) Einstein's religious beliefs did not allow him to fully understand the theory of quantum mechanics. 11. Which of the following can not be properly inferred based on the information in the passage? (a) The location of a particle within a closed box cannot be known for certain without observing the particle. (b) Properties such as position and energy of a particle can never be measured (0) Particles can be properly described as quantum mechanical waves. (d) all of the above (Q.12 - Q.14): Nineteenth century painter Albert Bierstadt'’s view of his artistic skill as a vehicle for self- promotion is was evident in his choices of style and subject matter. From the debut of his career with the exhibition of Lake Lucerne (1856), he developed a fixed style that was most easily recognizable for its size—the largest of the 636 paintings on display at the exhibition, it was over three meters wide. This, coupled with the artist's ability to represent the optimistic feeling in America during the westward expansion, is what led to Bierstadt’s explosive growth in popularity during the 1860's. Bierstadt deliberately appealed to those rich patrons—railroad tycoons and financiers—whose nearest substitute to making the arduous journey out West was to purchase a hyperbolized replica of a Western vista, But trends following the Civil War produced a drastic shift away from the adventurous optimism of the pre- War era and toward a more subdued appreciation for the details of American life, In this new social context, the paintings now seemed too decadent, too gaudy, for the new philosophy taking root in the country following the horrors of war. As one commentator in 1866 put it, Bierstadt’s work “may impose upon the senses, but does not affect the heart.” In a sense, then, that same American pride upon which Bierstadt had capitalized to advance his success was now, in its fickleness, the source of his downfall 12, According to the passage, the new “philosophy” taking root in America after the Civil War would be best described as (a) justifiable pessi (©) restrained minim ism (b) somber realism m (d) prideful idealism 13, The passage makes use of the phrase in quotations primarily in order to (a) challenge a prevailing thesis (b) point out an erroneous assertion (c) provide expert testimony (d) offer evidence supporting a claim 14, Allof the following are mentioned as contributors to Bierstadt’s success EXCEPT: (a) the dimensions of his paintings (b) his ability to convey auspicious feelings (c) subdued appreciation for the details of American life (d) catering to the preferences of the wealthy mock cLaTits 4 LEGALEDGE Directions (.15- Q.29): Read the following passage carefully and answer the questions given below it. Certain words in the passage are printed in bold to help you to locate them while answering some of the questions. Globalisation, liberalisation and free market are some of the most significant modem trends in economy. Most economists in our country seem captivated by the spell of the free market. Consequently, nothing seems good or normal that does not accord with the requirements of the free market. A price that is determined by the seller or, for the matter, established by anyone other than the aggregate of consumers seems pernicious. Accordingly, it requires a major act of will to think of price-fixing as both normal and having a valuable economic function. In fact, price-fixing is normal in all industrialised societies because the industrial system itself provides, as an effortless consequence of its own development, the price-fixing that it requires. Modern industrial planning requires and rewards great size. Hence a comparatively small number of large firms will be competing for the same group of consumers. That each large firm will act with consideration of its own needs and thus avoid selling its products for more than its competitors charge is ‘commonly recognised by advocates of free-market economic theories. But each large firm will also act with full consideration of the needs that it has in common with the other large firms competing for the same customers. Each large firm will thus avoid significant price-cutting, because price-cutting will be prejudicial to the common interest in a stable demand for products. Most economists do not see price-fixing when it ‘occurs because they expect it to be brought about by a number of explicit agreements among large firms; itis not Moreover, those economists who argue that allowing the free-market to operate without interference is the most efficient method of establishing prices have not considered the economics of non-socialist countries. Most of these economies employ intentional price-fixing, usually in an overt fashion. Formal price-fixing by cartel and informal price-fixing by agreements covering the members of an industry are common place. Were there something peculiarly efficient about the free market and inefficient about price-fixing, the countries that have avoided the first and used the second would have suffered drastically in their economic development. There is no indication that they have Socialist industry also works within a framework of controlled prices. In the early 1970's the Soviet Union began to give firms and industries some flexibility in adjusting prices that a more informal evolution has accorded the capitalist system. Economists in the USA have hailed the change as a retum to the free- market. But the then Soviet firms were not in favour of the prices established by a free-market over which they exercised little influence; rather, Soviet firms acquired some power to fix prices. 15, The author's primary objective of writing the passage seems to (a) belie the popular belief that the free-market helps enhance development of industrial societies (b) advocate that price-fixing is unavoidable and it is beneficial to the economy of any industrialized society (0) explain the methodology of fixing price to stabilise free-market (d) create awareness among the general public regarding combating price-fixing by large firms 16. Which of the following statements (A), (B) and or (C) is/are true in the context of the information given in the passage? The information in the passage is helpful to (A) know some of the ways in which prices can be fixed (8) identify the products for which price-fixing can be more beneficial (C) differentiate between the economics of various countries (@) Only (A) (b) Only (B) (0) Only (C) (d) Only (A) and (8) 47. Considering the literal meaning and connotations of the words used in the passage, the author's attitude towards "most economists" can best be described as (a) derogatory and antagonistic (b) impartial and unbiased (©) spiteful and envious (d) critical and condescending mock cLaTits 4 LEGALEDGE 18, The author feels that price fixed by seller seems pernicious because (a) people don't have faith in large firms (b) people don't want the Government to fix prices (c) most economists believe that consumers should determine prices (d) most economi sts believe that no one group should determine prices 19. Which of the following stalements is definitely true in the context of the passage? Price fixing is () a profitable result of economic development (b) an inevitable result of the industrial system (0) the joint result of a number of carefully organised de (d) a phenomenon uncommon to industrialised societies 20. According to the passage, price-fixing in non-socialistic countries is generally (a) intentional and widespread (b) illegitimate but beneficial (0) conservative and inflexible (d) legitimate and innovative 21. What was the result of the then Soviet Union's change in economic policy in the 1970's?" (a) They showed greater profits (b) They had less control over the free-market (c) They were able to adjust to techno-advancement (d) They became more responsive to free-market 22. The author's primary concer seems to (a) summarise conflicting viewpoints (b) make people aware of recent discoveries (©) ctiticise a point of view (d) predict the probable results of a practice 23. Which of the following statements about the socialist industry is/are false? (A) It works under certain price restrictions (8) Ithas no authority to determine prices (C) Ithails the strategy of price fixing, as a major deviation (a) Only (A) is false (b) Only () is false (c) Only (C) is false (a) (A) and (B) are false 24, Choose the word which is most opposite in meaning of the word given in capitals as used in the passage. PERNICIOUS, (a) harmful (b) deadly (©) promotive (d) extravagant 25. Choose the word which is most opposite in meaning of the word given in capitals as used in the passage. CAPTIVATED (@) repelled (b) seized (0) tamed (d) enchanted 26. Choose the word which is most opposite in meaning of the word given in capitals as used in the passage OVERT (a) unambiguous —_(b) concealed (0) manifest (d) inexplicable 27. Choose the word which is most nearly the same in meaning as the word given in capitals as used in the passage. STABLE () uniform (b) fluctuating (©) permanent (d) restored mock cLaTits 4 LEGALEDGE 28. 29. Choose the word which is most nearly the same in meaning as the word given in capitals as used in the passage. EXPLICIT (@) clean (b) implied (c) obvious (4) specifically Choose the word which is most nearly the same in meaning as the word given in capitals as used in the passage, ADVOCATES (@) lawyers (b) experts (c) recommends (4) supporters mock cLaTits 4 LEGALEDGE Directions (@.30-Q.68): Read the information given below and answer the questions based on it. (Q.30-Q.35): The dispute over Mahadayi river has resurfaced over the past few weeks and tensions have been rife between [1] and [2]. The dispute over Mahadayi river began in the 80s and grew stronger in the subsequent decades. The trigger was [2]'s move to design a number of dams, canals and barrages to route the Mahadayi river water to the Malaprabha basin. The state claimed that channelling the river water into the basin of Malaprabha, a tributary of the [3], would meet the requirements of its water-scarce districts. [1], seeking redressal to the dispute in 2002, sought the constitution of a water disputes tribunal. The state also moved the apex court in 2006 with its demand. After sustained efforts by the [1] government, the Mahadayi Water Disputes Tribunal was set up on November 16, 2010. 30. Which of the following states will replace [1] and [2] respectively? (a) Goa and Karnataka (b) Karnataka and Andhra Pradesh (©) Andhra Pradesh and Tamil Nadu (a) Tamil Nadu and Kerala 31. Which of the following will replace [3]? (@) Godavari (b) Krishna (©) Tapti (d) Kaveri 32. Which of the following projects is being constructed by [2] on the Mhadei river? (a) Kalasa-Banduri Nala Project (b) Belagavi-Bagalkot Project (0) Bhima-Dindi Nala Project (d) Venna-Koyna Nala Project 33. Which of the following articles of the Indian Constitution provides the Parliament for the adjudication of any dispute or complaint with respect to the use, distribution or control of the waters of, or in, any inter-State river or river valley? (a) 260 (b) 261 (©) 262 (d) 263 34. Other than [1] and [2], which other states does the Mhadei river flow through? (2) Kerala (b) Andhra Pradesh (c) Telengana (a) Maharashtra 35. Which of the following wildlife sanctuaries is the source of the Mhadei river? (a) Bhimgad Wildlife Sanctuary (b) Bhadra Wildlife Sanctuary (0) Sharavati Valley Wildlife Sanctuary (d) Brahmagiri Wildlife Sanctuary {Q.36-.40): [1] has been approved as an observer state for a five-nation grouping in the Western Indian Ocean, which includes [5], Comoros, Seychelles, Mauritius and French Reunion. The members of the [3] decided on [1] application at a meeting last week, officials said. With the decision, [1] will join [2], which was made an observer in 2016, as well as the “International Organisation of the Francophonie” or the 54-nation French-speaking collective, the European Union (EU) and Malta, which were all admitted in 2017. The decision to join the (3] marks a part of the government's push for greater salience in the whole Indian Ocean Region (IOR), including what is called the Westem or African Indian Ocean. In December 2019, the Ministry of External Affairs decided to include Madagascar, Comoros and Reunion as part of the IOR (Indian Ocean Region) desk along with Sri Lanka, Maldives, Mauritius and Seychelles. The [3] is also significant for its geographical location, as the islands sit around a “key choke-point” in the Indian Ocean — the Mozambique Channel. This channel is being watched more closely as the U.S.-Iran tensions threaten the Strait of Hormuz. Given China’s growing presence in the region, [1] hopes to increase its naval presence and gain support for its maritime projects across the Indo-Pacific, beginning at East African shores. 36. Which country has been replaced with [1] in the above passage? (2) Srilanka (b) India (c) Maldives (6) Vietnam mock cLaTits 4 LEGALEDGE 37. Which country can be replaced with [2] in the above passage? (@) Mauritius (b) Seychelles (0) India (4) China 38. Which organization can be replaced with [3] in the above passage? (a) International Maritime Organization (b) South Asian Ocean Committee (0) South East Asia Ocean Committee (d) Indian Ocean Commission 39. In which year the organization referred by [3] was set up? (a) 1982 (b) 1983 (©) 1990 (d) 1989 40. Which country can be replaced with [5] in the above passage? (@) Seychelles (b) Singapore (0) Srilanka (d) None of the above {Q.41-0.45): The decision by the WADA to ban [1] from global sporting events for a four-year period is arguably the biggest sporting crisis the country has faced till date. This move will hurt [1] the most at the 2020 Tokyo Olympic Games and the 2022 Beijing Winter Olympics where the nation’s flag, name and anthem will not be allowed, [1] will inevitably approach the Court of Arbitration for Sport with an appeal, but if the sentence is upheld it could bar the nation from participation in several high-profile global sporting events including the 2022 football World Cup. The saga has its roots in the scandal that erupted on the eve of the 2016 Rio Olympics, when whistle-blower reports nailed [1] for running one of the most sophisticated doping programmes. The allegations centred around the active collusion of [1]n anti-doping experts, the sports ministry and members of the country's intelligence service in replacing dope-tainted urine samples. with clean ones during the 2014 Winter Olympics in Sochi, In September 2018, as part of the resolution of that case, [1] reluctantly agreed to open up its database to corroborate the findings of the reports. WADA has now ruled that the country manipulated this very database in order to cover up large-scale violations. However, ahead of the Rio games, Russia had recommended that [1] be expelled, but the International Olympic Committee (IOC), under President Thomas Bach, had left the decision to individual sports’ governing bodies, and, subsequently, athletes who were cleared of doping were allowed to compete as neutrals. 41. 2022 Football World Cup will be the ___ edition of the World Cup. (@) 20th (b) 21st (©) 22nd (4) 23rd 42. In the above passage what has been redacted by [1]"? (@) Russia (b) China () North Korea (d) Japan 43, Where is the headquarters of the WADA situated? (@) Lausanne (b) Montreal (0) Paris (d) Geneva 44. The International Olympic Committee was found in __ (a) 1944 (b) 1916 (o) 1894 (d) 1780 45. How many medals did India win in the 2016 Rio Olympics? (a) 25 (b) 18 (11 (a) 2 (Q.46.2.49): [1] will receive the Order of Merit for his conduct as leader of the ship that was quarantined for nearly a month. He was the last to leave the ship just a few days ago The Italian captain of the Diamond Princess cruise ship will be given his country’s highest honor, the Order of Merit, by President [2], the president's office confirmed, [1] has been widely praised for the care and concern he gave to the crew and passengers on the quarantined cruise ship, and for being the last person to leave the ship. [1] is stil in Japan, where he is in temporary quarantine until local health officials are sure he is not carrying the COVID-19 novel coronavirus. mock cLaTits 4 LEGALEDGE 46, Fill [1] with the suitable option? (a) Gennaro Arma (b) Marianna Arma (c) George gill (d) Leonardo Vans 47. The Japanese authority imposed a quarantine process that would see __ passengers test positive for the virus (@) 700 (b) 725 (0) 755 (4) 705 48. coronavirus disease (COVID-19) that was first reported from Wuhan, China, on __ 20197 (a) January 25" (b) February 25" —(c) December 31" (d) February 1st 49, Fill [2] with suitable option. (a) Sergio Mattarola (b) Emmanuel Macron (6) Angela Merkel (d) Marine Le pen (Q.50-Q.54): Medical details of over 120 milion Indian patients have been leaked and made freely available on the Internet, according to a recent report published by [1], a German cybersecurity firm. What is even more worrying is that the number of data troves containing this sensitive data went up by a significant number in the Indian context a month after [1] initial report was published. The updated report also places [2] at the top of the States affected by the leak. The first report was published in October last year, in which [1] revealed a widespread data leak of a massive number of records, including images of CT scans, X-rays, MRIs and even pictures of the patients. The follow-up report, which was published in November, classifies countries in the three categories based on the action taken by their governments after the first report was made public. India ranks second in the “[4l’ category, after the U.S. The report says that in 60 days after the first report was put out, the number of data troves bearing the patients’ information went up from 6,27,000 to 1.01 million, and that the images of patients’ details rose from 105 million to 121 million “Itis a notable fact for the systems located in India, that almost 100% of the studies (data troves) allow full access to related images,” the report states, As per the follow-up report, [2] ranks the highest in terms of the number of data troves available online, with 3,08,451 troves offering access to 6,97,89,685 images. The next is Karnataka, with 1,82,865 data troves giving access to 1,37,31,001 images. “The leak is worrying because the affected patients can include anyone from the common working man to politicians and celebrities. In image-driven fields like politics or entertainment, knowledge about certain ailments faced by people from these fields could deal a huge blow to their image. The other concem is of fake identities being created using the details, which can be misused in any possible number of ways,” a [2] cybersecurity officer said Medico-legal expert Lalit Kapoor said any communication between a doctor and a patient was privileged ‘one. "A doctor or a hospital is thus ethically, legally and morally bound to maintain confidentiality,” he sai PACS servers [1] original report says the leak was facilitated by the fact that the Picture Archiving and Communications ‘Systems (PACS) servers, where these details are stored, are not secure and linked to the public Internet without any protection, making them easily accessible to malicious elements. 50. Which firm can be replaced by [1] in the above passage? (a) Greenhouse Sustainable Resilience (b) Greenbone Sustainable Resilience (c) Green Sustainable Resilience (d) Greenhood Sustainable Resilience mock cLaTits 4 LEGALEDGE 51. Which state can be replaced by [2] in the above passage? (@) Maharashtra —_(b) Kerala (0) Telangana (d) Karnataka 52. Which of the following bill has been tabled in Indian parliament to secure personal data of citizens? (a) The Personal Data Protection Bill, 2019 (b) General Data Protection Regulation (c) Data Security Bill, 2019 (d) Critical Data Protection Bill, 2019 53. In which category India has been placed in the above said report? (a) Good (b) Bad (©) Ugly (d) Best 54, Which landmark judgment held the right to privacy to be the part of Article 21 of the Constitution of India? (a) K.S. Puttaswamy (Retd) vs Union of India (b) Navtej Singh Johar v. Union of India (c) Olga Tellis v. Bombay Municipal Corporation (d) Gurbaksh Singh v. Union of India (Q.55-Q.58): The US and Taliban signed an agreement for “Bringing Peace to Afghanistan’, which will enable the [1] to withdraw troops in the next [2] months, India attended the signing ceremony in Doha, and was represented by Ambassador to Qatar P Kumaran The pact is between the [3] which is not recognized by the United States as a state and is known as the Taliban’ and the US. The four-page pact was signed between Zalmay Khalilzad, US Special Representative for Afghanistan Reconciliation, and Mullah Abdul Ghani Baradar, political head of the Taliban. While the agreement creates a path for the [1] to gradually pull out of its longest war, many feel that the talks, scheduled to take place between the Afghan sides, could be much more complicated. The Afghan government has been completely sidelined during the talks between the US and Taliban. The future for the people of Afghanistan is uncertain, and will depend on how Taliban honours its commitments and whether it goes back to the mediaeval practices of its 1996-2001 regime. 55. Which of the following will replace [1]? (a) US only (b) NATO only (0) Both US and NATO (d) Allied 56. Which of the following will replace [2]? (a) 10 (b) 12 (9) 14 (d) 16 57. Which of the following will replace [3]? (a) Islamic Caliphate of Taliban (b) Taliban Afghan Caliph (0) Islamic Emirate of Afghanistan (d) Emirate of Afghan Talibans 58. Which of the following statements is incorrect? (a) Within the first 135 days of the deal the forces in Afghanistan will be reduced (b) The UN will lift sanctions on the Taliban (c) The US will lft sanctions on the Taliban (d) The deal has also provided for prisoner swap mock cLaTits 4 LEGALEDGE (Q.59-Q.63): Finding the “missing millions” has always been the biggest challenge in achieving the global goal of ending tuberculosis (TB) by Of the estimated 4.3 million people who are “missed” by health care systems, India, as per successive global TB reports, alone has one million “missing” However, as per Prime Minister Narendra Modi’s commitment, india has set its sights on achieving the End TB goal five years earlier than the global deadline Ina dramatic leap towards that aim, India has managed to trace 70 per cent — around 7 lakh TB patients — in the last year, with a combination of incentives for doctors, expansion of diagnostic network, nutrition support for patients and a legal provision to punish non-notification of patients by private sector doctors, in 2018, India passed an order that said failure by clinical establishments to notify a tuberculosis patient to the nodal officer and local public health staff can be punished with a jail term of six months to two years. However, the provision, officials say, is sparingly used as the idea is not to harrass. The National Strategic Plan for TB Elimination 2017-[1] says, “India has been engaged in TB control activities for more than 50 years. Yet TB continues to be India’s severest health crisis. TB kills an estimated 480,000 Indians every year and more than 1,400 every day. India also has more than a million ‘missing’ cases every year that are not notified and most remain either undiagnosed or unaccountably and inadequately diagnosed and treated in the private sector.” 59. Which bacteria is responsible for causing TB? (@) Actinobacteria tuberculosis (b) Mycobacteriaceae tuberculosis (c) Mycobacterium tuberculosis (d) None of the above 60. In which city National Tuberculosis Institute is situated? (@) Dethi (b) Mumbai (0) Bangalore (d) Hyderabad 61. What is the deadline for National Strategic Plan for TB Elimination — [1]? (a) 2022 (b) 2023 (c) 2024 (4) 2025 62. On which day the world TB day is celebrated? (@) 24th March (b) 25th March (0) 24TH April (4) 25th April 63. Where was the first ever high-level meeting on tuberculosis held in? (a) New York (b) Geneva (©) London (d) Washington (Q.64-0.68): The Indian Air Force (IAF) and the [1] have joined forces to commence the fifth edition of Exercise Indradhanush EX Indradhanush — V 2020 is being conducted at Air Force Station Hindan, India, and will conclude on 29 February. The theme of this year's joint exercise is ‘Base Defence and Force Protection’. It highlights the recent terror threats to military establishments. Indradhanush will allow the IAF and [1] to strategise and share information, as well as learn from each other's operational experience. Through this exercise, the two airforces can plan scenarios and train on tactics to counter terror threats and protect their installations. Additionally, the forces will share their training philosophies and best practices along with contemporary technologies. The [1] team will include 36 specialised combatants of the [1] Regiment, whereas the IAF team will ‘comprise 42 combatants of the GARUD Force. 64. Exercise Indradhanush was last conducted in__ in UK. (a) 2018 (b) 2017 (9) 2016 (4) 2015 65. Fill [1] with suitable option, (a) Air Force One (b) Royal Canadian Air Force (0) British Royal Air Force (d) Russian Air Force mock cLaTits 4 LEGALEDGE 66. 67. 68. Who was the first Indian Air Force chief? (@) Birender Singh Dhanoa (b) Subroto Mukerjee (0) Rakesh Kumar Singh Bhadauria (d) Arup Raha The Air Force Day is celebrated every year on __, the day Indian Air Force was officially established in 1932. (a) October 8 (b) December 21 (c) January 3 (4) April 2 Who among the following is the current Defence Minister of India? (a) Rajnath Singh (b) Amit Shah (0) Harsimrat Kaur Badal (d) Santosh K Gangwar mock cLaTits 4 LEGALEDGE Directions (Q.69-Q.72): The Calcutta High Court recently observed that courts must consider practical “hard realities" before they decline to grant equitable relief citing delay in approaching the Court. As stated in the order passed by a Division Bench, "The legal principle ‘delay defeats equity’ is not abstruse nor to be perceived theoretically, but must be applied with the hard realities of today’s world and above all objectively. It is not a stray subjective consideration, but has an in-built objective value, which should always be borne in mind when a right or relief is denied on such equitable principle.” The trial court, in this case, had declined to grant an ex-parte interim injunction citing the "considerable delay" on the litigant's part in approaching the court, about a month after the cause of action had arisen The relevant portion of the impugned order read, "The injunction is an equitable relief and there is well known maxim "delay defeats equity’. So considering the same, | am not inclined to pass any order of ad- interim injunction at this stage. Disagreeing with the trial court's stance, the High Court highlighted that the trial court had erred in presuming that the litigant had shown a dormant attitude in approaching the Court. In doing so, the Court pointed out that itis improbable that a lawyer would not take at least fifteen to twenty days to take a decision and institute the suit before the Court of law This being the case, the Court held that the challenged trial court order was testament to fact that the learned Judge had "shitked his responsibility in recording the reasons on merit and had invented a circuitous path in refusing the prayer for ex-parte ad interim order of injunction." While the High Court was inclined to interfere with the trial court order, the judges observed that the application for interim injunction has now been contested by the opposite parties. This being the case, the Court proceeded to take critical note of the far-off date fixed for the next hearing in the matter i.e. August 2020, The Bench observed that such a delay in hearing the matter would frustrate the object of filing the injunction application. The Bench observed, “If the longer date is fixed for disposal of an application for injunction, it frustrate the main object of it and it is high time that the Judges should realize that the purpose of incorporating the provisions for temporary injunction is not only to protect and preserve the status of the property, but also to protect the rights pending final adjudication in the suit.” The trial court was, therefore, directed to prepone the date of hearing, if approached by either of the parties. Further the lower court was also directed to make efforts to dispose of the same as expeditiously as possible without granting unnecessary adjournments. 69. Mrs, Samiksha, purchased the product of "Glowing Face” company for her facial wrinkles. She realised after two months using that product, that it had in turn damaged her skin. She gave the product to her scientist friend who conducted laboratory test on the product. The findings of the test shows that product used some chemicals which is harmful for the skin and thus after three months of coming out this report she approached court of law demanding injunction order against the company, According to the information given in the passage, would court accept her plea? (@) Court will accept her plea as they will consider the hard realities of today's world and thus understand the reason of the delay. (b) Court will not accept the plea as Mrs Samiksha is very late and thus delay defeats equity principle will hold and there is no reason specified justifying delay. (c) Court will accept her plea because she has not intentionally caused delay in filling the suit. (d) Court will not accept her plea because she has intentionally caused delay in filing the suit. mock cLaTits 4 LEGALEDGE 70. A cause of action was brought against Malwa Manufacturing Plant, one of the biggest plants of the area It has been argued that the plant uses old technology and thus discharges huge amount of waste in the water bodies, Due to this life in nearby villages have been impacted and studies shows that percentage of people having health issues are increasing. The cause of action has been collectively brought by NGOs after conducting the studies of the area and they are demanding for injunction order against the plant. However, court dismissed the plea by saying that plant has been functioning in the same way since last ten years and since then nobody complaint about it. (@) The Court is right in declining the plea as considerable time has been passed and thus plea is not adimissible. (b) The court is wrong in declining the plea as the plant is the people. (0) The court is right as there is unreasonable delay on the part of complainant to raise the matter and closing of the plant will have drastic impact on the employment conditions of the area. (d) The court is wrong because there is no unreasonable delay in reporting the matter considering the practical reality and time required to detect the problem. negatively impacting environment and health of 71. Mary and Tom have lived in their home for more than 40 years, and have cultivated a lovely property, filled with mature shade trees. When a new neighbour moves in, he begins making plans to put in a newer, wider driveway, which will necessitate the removal of a fence, and removal of, or damage to a very large oak tree that has grown up partially on both properties. Mary and Tom do not want the tree removed, and thus decides to file a civil lawsuit seeking a civil injunction preventing the neighbour from doing so. After two months, they filed lawsuit by the time their neighbour started his construction according to his plan. Will their plea be successful? (@) They will be successful as the new property will destroy their home and thus they will get relief from injunction order. (b) They will not be successful as there is no reason for court to stop a person carrying legal construction in his property. (0) They will be successful as there is no reasonable delay on their part and thus, they can save their home from injunction order. (d) They will not be successful as there is reasonable delay and thus granting them injunction would cause loss to the neighbour as he had already began his construction work 72. After Nick is fired from his job from R&R consultancy services, and at the time of leaving, he threatens to disclose corporate secrets to the public. Because disclosure of sensitive company information could cause serious damage, the corporation decides to file for an injunction to keep Nick from talking about the ‘company’s secret. It was the clause in employment contract of Nick and R&R consultancy services that after completion of his services, he will not work in similar firm for one year. Nick left his job in May 2019, and thus in June 2020, he started working with A and B major rival firm. R & R file for suit on June 2020, as soon as they got to know about his new job? (a) R &R cause reasonable delay in filing suit as he left job in 2019 and thus cause of action begins then (b) R & R did not cause reasonable delay as they file for a suit of injunction as soon as they came to know about Nick's new employment (c) R &R cause reasonable delay in filing the suit as by this time he might have leaked firm's secret. (d) R & R did not cause reasonable delay as according to terms of the contract he could not work for one-year thus there is no scope of leaking secrets. mock cLaTits 4 LEGALEDGE (Q.73-Q.76): The Supreme Court recently issued a ruling that reservation in jobs and in promotions is not a fundamental right, The case pertains to a decision by the Uttarakhand government in 2012, Back then, the government had decided to full up posts in public services without providing reservation to members of the Scheduled Caste (SC) and Scheduled Tribe (ST) communities. The Uttarakhand High Court directed the state government on July 15, 2019 to implement reservations in promotion by promoting only SCs and STs to maintain the quota earmarked for the said categories. In a review petition filed by the Uttarakhand government, the High Court made three critical observations. Firstly, the Court said that Article 16(4A) of the Constitution is an enabling provision that does not vest a right to claim reservation in any group of persons. In other words, the state is not under an obligation to provide reservation to members of Scheduled Castes and Scheduled Tribes. Secondly, the High Court maintained that if the government wants to implement reservation, it has an obligation to justify its decision through quantifiable data which shows that members of Scheduled Castes and Scheduled Tribes are not adequately represented in the services of the State. Thirdly, the Court directed the government to collect the relevant quantifiable data which would facilitate a decision on whether to grant reservation of not, The High Court stressed that the government can take a decision about granting reservation only after collecting the quantifiable data. The Supreme Court, in its decision, agreed with the first two observations of the High Court and overruled the third one. The Apex Court has affirmed that reservation cannot be claimed as a matter of right and that the reservation provisions in Article 16 are merely enabling in nature. It is also well settled that without quantifiable data showing inadequate representation, the government cannot grant reservation, Here, the Court has further emphasized that the government is not under a similar obligation to collect data while deciding not to implement reservation. Essentially, the Court has absolved the government from any kind of answerability when the government decides not to exercise a discretionary power. The Supreme Court said that a writ of mandamus cannot be granted by any court in order to enforce an enabling provision. The writ of mandamus is issued only to compel an authority to discharge a binding duty. Thus, the High Court was wrong in directing the state government to collect data in order to decide on the issue of reservation 73. In the state of Airan, the tribe Rajdohi is a minority community. They have been long discriminated against by the majority community due to their peculiar tribal practices and the locational disadvantage that this community suffers because of being concentrated in the fringes of the forest. Avani, a Rajdohi tribal member fought against all odds to complete her schooling in the city. She secured exemplary grades in her graduation. However, despite three attempts at the State Service Exams she could not clear it. The State Service Exams do not grant reservation to the Rajdohi tribe, Avani filed a wrt petition under Article 32 alleging the violation of her fundamental right (a) She has to first move the High Court before approaching the Supreme Court. (b) The Supreme Court will issue a writ of Mandamus directing the State to grant reservation to the Rajdohi Tribes. (0) The government is obliged to issue a justification stating why they have excluded Rajdohi tribe from reservation in the State Service Examination. (d) None of the above mock cLaTits 4 LEGALEDGE 74, In the state of Airan, Rajdohi is a tribal community. This tribe however is not identified as the ‘Scheduled tribe’ under the constitution. They have been long discriminated against due to their peculiar tribal practices and the locational disadvantage that this community suffers because of being concentrated in the fringes of the forest The newly elected government of the state was headed by Avanish, a member of the Rajdohi tribe. Soon after his election the State rolled out a policy reserving 30% of the seats in the State Services exam for the Rajdohi Tribes. Parag Pander, a member from the RSS, filed a petition before the High Court challenging the government's decision (@) The decision of the government is not based on any quantifiable data therefore will be quashed as it is a mere subterfuge by the leader who belongs to the same community. (b) The percentage of reservation granted is 30% which falls within the cap of 50% as was declared by the Supreme court in its Indra Sawhney judgment. Therefore, the reservation is constitutional. (6) The government does not have to show any quantifiable data in this case as the tribe is not a scheduled tribe. (d) The government is obliged to provide a quantifiable data showing the inadequate representation of the community. 75. Strictly based on the understanding of the passage complete the following sentence, A decision of the state to grant reservation to the community X and exclude Y from the reservation provisions of the State services ‘exam has to be: (a) Backed by a quantifiable data expressing the inadequacy of representation community X in the State services (b) Backed by a quantifiable data expressing the adequacy of representation of community Y in the State services (c) Both (a) and (b) (d) Cannot be granted reservation unles: Caste” as given under the Constitution falls within the meaning of “Scheduled Tribes or Scheduled 76. Mahela, is a member of the Scheduled Caste. The State introduced services in the field of airplane aviation in 1975. Under the introduced services, pilots were to be scouted on the basis of the physical requirement and a minimum of 90% in their metric qualification. In the history of the aviation services, there has been no officer belonging to the Scheduled caste category and so far only 4 women have been appointed as aviation officers. The state brought about an amendment reserving 50% seats for women. Mahola applied for the services. She passed the physical requirement test but was rejected on the basis that she has secured only 89.5% in her metric qualification. Mahela filed ‘a writ petition in the High Court alleging that the State should relax the provision as she is a member of the disadvantaged Scheduled Caste category and meets the physical requirement and moreover, she is only short of 0.5% to pass the 90% mark of educational qualification. (@) Given the fact that there is no representation of the SC category and the candidate is also a woman, the state should allow a relaxation for Mahela. (b) The services pertain to aviation and hence fall outside the purview of state reservation scheme (c) Itis the discretion of the state to grant reservation to such candidates (d) None of the above mock cLaTits 4 LEGALEDGE (Q.77-Q.80): There might be no question that Article 48 of India’s constitution requires the state to endeavour to organise agriculture and animal husbandry on modemn and scientific lines, in particular by taking steps for preserving and improving the breeds, and prohibiting the slaughter of cows and calves and other milch and draught cattle. This article, however, placed as it is in Part IV of the constitution, which contains the directive principles of state policy, is not justiciable in law. In other words, while a duty is cast on the state to make laws based on the urging of the various directive principles, no person can question the state in a court of law on the ground that a provision in Part IV stands violated by either the government's, action or inaction, as the case may be, Moreover, in any event, the duty imposed by Article 48 has to be read in conjunction with the federal structure that India’s constitution imposes. The Seventh Schedule to the document, which contains three separate lists, defines the differing roles of governance respectively played by the Union and the state governments. On issues contained in List 1, the constitution provides that parliament alone can make laws; on issues mentioned in List 2, itis the state governments that have the exclusive power to make and enforce legislation; and, on issues contained in List 3, both the central and the state government can legislate, subject to the caveat that parliamentary law would prevail in the case of any conflict, except in certain limited circumstances. Under this design, by virtue of Entries 14 and 15 of List 2, itis the states alone that have been granted the power to make and enforce laws on issues concerning agriculture and the “preservation, protection and improvement of stock.” 77. The Central Government passed a legislation banning cow slaughter as it goes against the religion of Hindus. Moreover, the legislation aims for the preservation and protection of the stock, Mohammad Hanif filed a petition in the court challenging the legislation. (a) The legislation is likely to be struck down as it goes against the provisions of the Constitution (b) The legislation is aimed to protect the religious interests of the Hindu community therefore the court will uphold it and is in consonance of Article 48. (0) None of the given options. (d) The legislation goes against the interest of Muslim community and should be struck down. 78. In the state of Uttar Pradesh, the instances of cow slaughter suddenly spiked that resulted in a slump in the dairy industry. In the interest of the milch cattle, the state passed a legislation regulating cow slaughter and only allowing slaughter of such cattle that are sterile or have crossed the age of fertility. The Haneef muslim religious group filed a petition alleging violation of their rights. (a) The legislation is unconstitutional as it goes against Islam. (b) The legislation should be struck down as Article 48 is only a discretion and not a binding obligation. (0) The state has the power to legislate on issues related to agriculture. Moreover, the legislation imposes only reasonable restrictions and hence is valid (d) None of the above. 79. Inthe state of Uttar Pradesh, the instances of the instances of cow slaughter suddenly spiked that resulted in a slump in the dairy industry. There were protests all across the state by the dairy industry imploring the state to intervene in the situation, Consequently, a petition was filed alleging that the state has failed to observe its duty under Article 48. (a) The state shall intervene in the situation (b) Under article 48, the state is under no obligation to make a legislation as it is only a directive principle, (c) The state shall be directed to ban cow slaughter. (d) None of the above. 80. Article 44, of the Constitution of India, provides for a Uniform Civil Code. However, even after more than 70 years of Independence the country has failed to pass a Uniform Civil code. In the wake of nation wide protests, the law commission recommended that it is time for the Legislature to pass a Uniform code that governs all the religion, all the states with a uniform civil system. However, even months after it the Government did not come up with any such legislation, Moreover, the situation kept getting worse as the protests flared, Anuj, a public-spirited individual filed a petition before the court. (a) The state is under an obligation to pass the Uniform Civil Code. (b) The court shall dismiss the petition as it refers to a religious matter. (c) The court can only make a recommendation as the State is under no obligation. (d) This is an emergency situation. The court will issue a writ of Mandamus directing the State to pass the uniform civil code. mock cLaTits 4 LEGALEDGE (Q.81-0.86): Road safety is a State subject. The administration of the Motor Vehicles Act, 1988 is under the transport Department. The aforesaid Act provides in detail the legislative provisions regarding registration of motor vehicles, For exercising the legislative provisions of the Act, the Government of India made the Central Motor Vehicles Rules 1989. Additionally, there are Rules of Road Regulations, 1989. The RTO is the one with which every individual is interacting, its services are discussed below: Law relating to Registration of Vehicle + Mandatory Registration: Under section 39 of the Motor Vehicle Act, 1988, it prohibits driving of any motor vehicle or any vehicle, which is not registered or no owner of vehicle should permit driving of any vehicle in public place which is not registered under the provision of the MV Act. Exception to this provision is cars with the dealers. + Jurisdiction for Application: Registration of the vehicle is done by the concerned authority is done on the basis of your residence or place of residence or place of business, where the vehicle is normally kept. + Application for Jointly owned Vehicle: In case of joint ownership, the registration of vehicle can be applied by one of the owners. If a vehicle registered in one state is kept in another state for more than a period of 12 months, then owner of such vehicle has to approach the registration authority for assignment of new registration mark within whose jurisdiction the vehicle is. + Change of Address: If there is change of address, then also, the owner of vehicle is required to approach the authority within 30 days in whose jurisdiction he has shifted for recording the change of address. Under section 48 of the motor vehicles act of 1988, the registering authority may, by order, refuse to register any motor vehicle, or renew the certificate of registration in respect of a motor vehicle (other than a transport vehicle), if in either case, the registering authority has reason to believe that itis a stolen motor vehicle or the vehicle is mechanically defective or fails to comply with the requirements of this Act or of the rules made thereunder, or if the applicant fails to furnish particulars of any previous registration of the vehicle or furnishes inaccurate particulars in the application for registration of the vehicle or, as the case may be, for renewal of the certificate or registration thereof and the registering authority shall furnish the applicant whose vehicle is refused registration, or whose application for renewal of the certificate of registration is refused, a copy of such order, together with the reasons for such refusal 81. John purchased a new sports bike. It hasn't received its registration number yet, But John got too excited to flaunt it before his friends, so he took it out for a Zoom, En route to a club, cops stopped him and seized the bike. John reasoned that his bike is new and the registration number is not yet issued, but the cops gave him no ear and asked him to appear before the court. According to the information provided in the above passage, was this action by the police correct? (a) Yes. Being an agency of the state, the police have absolute power to do so. (b) No. The bike has the provisional registration on i. This action by the police was not legal. (©) Yes. No owner of vehicle should permit driving of an unregistered vehicle in public place. (d) No. The police has no power to seize any vehicle 82, Karan Singh resides in New Delhi, He purchased a pre-owned Audi Q7 from Ranbir Kapoor of Chandigarh. its been eleven months since the purchase and the car still carries a Chandigarh registration. One day cops spot the car on the streets of Noida and issue a challan. Karan challenges this in the court of law and the judge orders the challan-issuing cop to pay penalty to Karan from his personal account. According to the information provided in the above passage, what do you think would be the reason for such ruling? (a) Because Karan stil had somo time to apply for a local registration (0) Because Karan had a VIP registration no. on his car (c) Because Noida falls in UP, not in New Delhi (d) None of the above 83. Babu bhaiya, Raju, and Shyam opened a garage in partnership. They decide to buy a tow truck to cater the requirements of customers and get a pre-used tow truck. But they are now stuck at to whose name they shall get it registered, as the RTO authority denied granting the registration under their business's name. According to the information provided in the above passage, what should they do? (a) Resell the tow truck as registration is denied. (b) They should buy a new tow truck for the sake of the business. (©) They should challenge RTO in the court of the law. (d) They should re-apply for registration under any of the partner's name. mock cLaTits 4 LEGALEDGE 84. Virat Kohli purchased a new modified car and approached the local authority for registration. But the person in charge denied the registration. Virat Kohli took this matter to the court of law and got the responsible officer suspended. According to the information provided in the above passage, what could be the reason behind such events? (a) Because the concerned officer didn't provide reason for the denial (b) Because Virat Kohli is a celebrity, the case was ruled to pacity him. (c) Because the concerned officer failed to prove that the car had stolen parts installed in it. (d) None of the above. 85. Based in Bhopal, Amit owned a house and a car. He got transferred to Indore, where he took his car along Six months later, his car gets involved in an accident. Though the accident wasn't his mistake, but the insurance company refuses to pay him and the cops denied releasing the vehicle without a penalty. According to the information provided in the above passage, what could be the reason behind such action? (a) Because Amit's car was parked in no a parking zone. (b) Because Amit's car was stil recorded at his old address of Bhopal (c) Because the damage wasn't covered by the insurance provider. (d) Can't say. It's a state matter. 86. According to the information provided in the above passage, what allows the transport department to exercise legislative provisions about registration of vehicles? (a) Motor vehicle Act 1988 (b) Rules of Road regulations, 1989 (0) The central motor vehicle rules, 1989. (d) Highway act 1999, (Q.87-Q.92): Right to Information (RT!) is an act of the Parliament of India which sets out the rules and procedures regarding citizens’ right to information. It replaced the erstwhile Freedom of Information Act, 2002. Under the provisions of RTI Act, any citizen of India may request information from a "public authority” (a "body of Government" or "instrumentality of State") which is required to reply expeditiously or within thirty days, In case of a matter involving a petitioner's life and liberty, the information has to be provided within 48 hours. The Act also requires every public authority to computerise their records for wide dissemination and to proactively catagorise certain information so that the citizens need minimum recourse to request for information formally. Under the RTI Act, 2005, Public Authorities are required to make disclosures on various aspects of their structure and functioning, This includes: (i) disclosure on their organisation, functions, and structure, (ii) powers and duties of its officers and employees, and (ii) financial information. The intent of such suo moto disclosures is that the public should need minimum recourse through the Act to obtain such information. If such information is not made available, citizens have the right to request for it from the Authorities. This may include information in the form of documents, files, or electronic records under the control of the Public Authority, The intent behind the enactment of the Act is to promote transparency and accountability in the working of Public Authorities. ‘Public Authorities’ include bodies of self-government established under the Constitution, or under any law or government notification. For instance, these include Ministries, public sector undertakings, and regulators. It also includes any entities owned, controlled or substantially financed and non-government organizations substantially financed directly or indirectly by funds provided by the government. This law was passed by Parliament on 15 June 2005 and came fully into force on 12 October 2005. Every day, over 4800 RTI applications are filed. In the first ten years of the commencement of the act over 17,500,000 applications had been filed. RTlis a legal right for every citizen of India. The authorities under the RTI Act 2005 are called quasi-judicial authorities. This act was enacted in order to consolidate the fundamental right in the Indian constitution ‘freedom of speech’. Since RTI is implicit in the Right to Freedom of Speech and Expression under Article 19 of the Indian Constitution, it is an implied fundamental right. mock cLaTits 4 LEGALEDGE 87. Jaswinder Singh holds a Canadian passport. At the age of 21, he came to India, to his ancestral village, for the first time to find himself surrounded by the corruption and scam. He finds that the funds allocated by the central government for the development of his village funds were misused and did not reach the ground. Frustrated from this, he files an RT! application but it was turned down. Based on the information provided in the passage, do authorities have the power to tum down an RT! application? (@) No. Under the Right to Information Act of 2005, all the public officials are compelled to provide any information sought by an RT! application. (b) Yes. Since Jaswinder wasn't an Indian citizen, he could be denied this right. (c) No. The authorities clearly misused their power in this case and should be penalised according to the law. (d) Yes. If the applicant is demanding a piece of sensitive information, the request can be denied. 88, Aman is facing a kidney failure problem and is prescribed to undergo dialysis once a week. Coming from a financially weak background, he requests the state to provide him with medical assistance which was denied. After few days, he requests again, but the result remains the same. Frustrated from the outcome, ‘Aman files an RTI application asking the reason for denial of his request. According to the information provided in the passage, within how many days the authorities are bound the reply? (a) Within 48 hours (b) Within 30 days (6) Within 60 days (d) Within 24 hours 89. A flyover is to be constructed to connect the congested old city to the railway station. But it is taking too long even after the project has been cleared by all the departments. Suspecting a foul-play, an RTI activist files an application asking the financial details of the project. But the concerned authorities didn't reply to him for the first six months, and later said that they are not liable to disclose financial details. According to the information provided in the given passage, were the turn of events correct? (a) Yes. The state authorities are not liable to disclose the financial details of any public project. (b) No. The authorities should have replied within 48 hours. Strict legal action should be taken against them. (c) Yes. Since the project was in PPP model, details need not be disclosed. (d) No. The authorities should have replied within 30 days. Action by law should be taken against them. 90. According to the information provided in the above passage, The Right to Information Act was enacted to bring transparency and accountability in the working of public authorities, Who qualify to be termed as ‘public authorities’ here? (a) The CEO of private steel firms. (b) The project managers of private firms writing software for defence (c) The contractor of an under-construction building owned by an individual. (d) The regional directors of Airport Authority. 91. Gopal operates a skill development NGO for women. A women welfare activist asks Gopal to make all his record public on the behest of Right To Information. But Gopal denies the request saying that since it is a Non-Governmental Organization, he is not liable to make any record public. Based on the information provided in the passage, what could we the best possible argument to support gopal's claim? (a) Gopal is right since his organisation does not collaborates with the government. (b) Gopal is right since his NGO is not a public sector undertaking (c) Gopal is right since he does not get any funding from the state. (d) All of the above. 92. The Right to Information is a fundamental right by law. However, it cannot be enjoyed by everyone. By the information provided in the passage, can you tell who do not qualify to enjoy this fundamental right? (a) Indian citizen living in the Middle east. (b) People living in India but not Indian citizens. (c) Indian citizens residing in southeast Asia (d) Indian citizen who fall under super-rich tax slab. mock cLaTits 4 LEGALEDGE {Q.93-Q.97): Section 375 of IPC: Section 375 of IPC defines rape as a criminal offence and states that a man is said to commit rape when he has sexual intercourse with a woman against her or without her consent or if she is a minor. Under Section 375 of IPC, just penetration is sufficient to constitute the sexual intercourse necessary to the offence of rape. A man is said to commit rape when he has sexual intercourse with a woman under any of the following situations: a. Against her will b, Without her consent, c, With her consent but the consent was obtained by putting her or any person close to her in fear of death or of hurt. d. With her consent but the man knows that he is not her husband and the consent was given because the woman believes that he is the man with whom she is or believes herself to be lawfully married , With her consent but at the time of giving such consent the woman was unable to understand the nature and consequences of her consent. f, With or without her consent, when the woman is below the age of 16 years Section 375 IPC Explanation: Under Section 375 of IPC, just penetration is sufficient to constitute the ‘sexual intercourse necessary to the offence of rape. A woman's consent to sexual intercourse includes unequivocal voluntary agreement or willingness to participate in the specific sexual act by words, gestures or any form of verbal or non-verbal communication. However, if a woman does not physically resist the act of penetration, it shall not be regarded as consenting to the sexual activity, Section 375 IPC Exception: The act of sexual intercourse will not be considered rape when it is done by a man with his own wife and the wife is not under fifteen years of age. A medical procedure or intervention shall also not constitute rape. 93. Arshia and Mohan got martied in the year 2019. Arshia was to turn 15 on 21* May 2020 and Mohan was already 21 years old, She never let Mohan come near her, however on 20" May, 2020 when both of them were staying at Arshia’s house, he forced himself on her by rubbing his genital over her clothes and stripped her of her clothes and tried to have intercourse with her. Naturally, Arshia started screaming thereby scaring Mohan who ran away apprehensive of Arshia’s relatives retaliating. Arshia files a suit for rape under Section 375 against Mohan, Decide:- (a) Mohan committed the offence of rape. (b) Mohan did not commit the offence of rape. (c) Marital rape is not an offence. (d) Arshia’s had given consent by sleeping on the same bed with him. 94, Jampoo was a git! (aged 19 years) who was married to Gampu (aged 23). They had been married for 6 months and had a regular life. One day they had a few friends over and everyone indulged into heavy drinking. After everyone had left Gampu asked Jampoo to get blindfolded in the bedroom. She obliged Gampu then called his friend who was waiting outside the house. The friend then had intercourse with Jampoo and she did not resist as she thought it was Gampu she was getting intimate with. ‘She opened her blindfold in a while and realised what was happening. She screamed and ran away. She filed a suit for rape under section 375 against the friend and Gampu the next day. Decide = (a) Gampu and his friend have committed rape. (b) Gampu solely is guilty for the offence of rape. (c) The friend solely is guilty for the offence of rape. (d) No-one is liable as there was consent. 95. Sylvia was in a relationship with Plath and both of them were adults, One day Plath invited Sylvia over to his house for dinner. He slipped Rohypnol (a sedative) into her drink which she consumed and as a result fell into a state of unconsciousness. When she woke up in the morning next to Plath and both were devoid of clothing. She realised that something was really off and confronted Plath who threw her out of his house She filed a suit for rape the very same day. Decide:- (a) Plath has committed offence of rape if Sylvia can prove penetrative intercourse occurred. (b) Plath has committed the offence of rape regardless whether there was intercourse or not (0) Sylvia gave consent by coming over to his house (d) Plath has committed no offence as he was her boyfriend mock cLaTits 4 LEGALEDGE 96. Dimpi was a maid at Aaloo’s house (Both adults). Aaloo always made advances towards her but never did anything extreme, However, one day when he was home during a holiday from work he locked the room from inside where she was cleaning and forced himself on her. She did not resist physically and there were no signs of that either. She however repeatedly was asking him to stop and also tried to yell for help when he had intercourse with her but he put his hand on her mouth. Next day she filed a suit for rape against Aaloo. Decide:- (a) Dimpi gave consent as she did not resist physically. (b) Aaloo committed the offence of rape. (c) Dimpi does not have any grounds to complaint as she is obliged to render any services Aaloo asks for (d) Dimpi should have seen it coming as he was already making advances and as she did not complain, she gave consent. 97. Koustav and Aakash were living in the same hostel room. They were students in the same college, They used to be friends in the first year and also had sexual relations, however due to differences arising out of professional life, they drifted apart. One day Aakash got heavily drunk and passed out on his bed. Koustav initially asked Aakash for consent however as he was not responding coherently, he took advantage of the. situation and had intercourse with Aakash. He also took pictures while engaged in the act and leaked them online, Next day Aakash realised what had happened and got extremely disturbed which also pushed him into depression, Aakash filed a suit for rape against Koustav. Decide: (a) Koustav committed the offence of rape (b) They used to have relations in the and hence there was implied consent. (0) Koustav tried to take permission and hence had no malicious intent thereby making him innocent of the offence. (d) Koustay did not committed the offence of rape. (Q.98-Q.103): Article 15 occurs in the Constitution of India under Part Ill that with fundamental rights of the citizens of India. Further article 14 to 18 fully deal with the various issues related to Right to Equality. Fundamental rights are applicable to all the citizens of the country universally irrespective of race, gender, place of birth, caste, religion. In case any citizen of India comes across a case of violation of fundamental Tights, he or she can approach the Supreme Court of India for ultimate justice. The fundamental rights mentioned in the Indian Constitution have their origin from different sources like England's Bill of rights, the United States Bill of Rights and France's Declaration of the Rights of Man Article 15 (1) and (2) prohibits the nation from discriminating any Citizen on the basis of any one or many of the aspects such as religion, race, caste, sex, place of birth and others, These articles make room that there will not be any restriction whatsoever to any person on the above ground in order to access and make use of public facilities and amenities like shops, restaurants, places of entertainment, and others. No person in India can be denied the use of tanks, wells, bathing ghats, roads and sides of public resort that are created and managed fully or in part from the state funds or have been dedicated for the use of the general public. Starting from Article 15 (3). the constitution of India attempts to deal with protective discrimination, Article 15 (3) makes it possible for the state to create special provisions for protecting the interest of women and children Article 15 (4) capacitates the state to create special arrangements for promoting the interests and welfare of socially and educationally backward classes of the society such as SC and STs, Article 15 (5) moves a step ahead and empowers the country to make reservations with regard to admissions in the educational Institutes both privately run and those that are aided or not aided by the government. From this rule, only the minority-run institutions such as the Madarsas are exempted. Therefore, we can say that Article 15 is the foundation stone for making reservations in the country. mock cLaTits 4 LEGALEDGE 98. Alex wants her son to study in a school run by a Hindu organization that focuses on Vedic studies. The school denies admission on the grounds of her religion, which is Christianity. Alex thinks this is unfair and decides to stand against this bias. According to the information provided in the passage, where should she file her complain? (a) Alex should report this to the magistrate of the town. (b) Alex should file a petition in the Supreme Court of India (0) Alex should raise a complaint with the Minority Welfare Department. (d) The school is run by private funds, Therefore, Alex has no right to challenge the decision. 99. Mr and Mrs Singh are celebrating their 25th wedding anniversary. They went to an upscale restaurant in ‘South Delhi but were denied entry because the husband and wife were wearing Kurta-pajama and sari respectively. The restaurant says that they have a strict dress code policy - a blazer or tux for males and a long gown or single piece dress for women. But Mr Singh took believed it to be a violation of Article 15 and dragged the restaurant in the court of law. However, his petition was outright rejected. Now, based on the information given in the passage, can you tell why this happened? (a) Because there was no violation of Article 15, or any other law. (b) Because this was a case of violation of Article 14, Mr Singh made a wrong judgment, (c) Because Mr Singh reported it in the high court. He should have moved to the Supreme Court directly (a) None of the above. 100. For better English education, Amit Kumar, who falls under a reserved category, wants his son to study in a school run by the Indo-Roman Catholic Organisation. The school, however, denies admission on the grounds of minorities. Amit Kumar files a petition in the Supreme Court against the act of the school, but gets denied. According to the information provided in the passage, can you tell why his petition was denied by the apex court? (a) Because he should have filed a police complaint first. (b) Because he should have reported this to the education ministry first. (c) Because the supreme court has more important issues to address. (d) Because, by the law, the school had the right to do so. 101. According to the information provided in the above passage, can you tell what does Article 15 (3) allow? (a) Caste-based reservation in higher education. (b) Reservation of seats for women in the local bodies. (c) Access to shops and hotels for all. (d) Access to religious places regardless of gender. 102. Rajan filed a petition in the Supreme Court claiming that by law, the allocation of seats for OBC in higher education should not be more than 9%, but the government of Rajasthan passed a bill to raise it to 12%, This is unconstitutional and should be scrapped, But the Supreme Court rules the petition to be non-judicial. Can you tell why the Supreme Court did this? (a) Because by the law, the state government can do that (b) Because parliamentary bills cannot be challenged in the court of law. (0) Because the bill should have been challenged in the high court (d) Correct reason is not mentioned in the list. 103. According to the information given in the passage, which of the following does not qualify as a violation of any of the provisions of Article 15? (a) Laxmi denied entry in Swami temple because the priest suspected she is on her periods. (b) Ramesh was told to keep out of an eatery because he works as a scavenger. (c) Aparna was forced to be a housewife even after being highly qualified (d) Roshni being denied a promotion even after being the best and deserving candidate because of her gender. mock cLaTits 4 LEGALEDGE Directions (Q.104-Q.106): Read and analyze the passage and attempt the following questions. in a typical Western liberal context, deepening of democracy invariably leads to consolidation of ‘liberal values.’ In the Indian context, democratization is translated into greater involvement of people not as individuals’ which is a staple to liberal discourse, but as communities or groups. Individuals are getting involved in the public sphere not as ‘atomized’ individuals but as members of primordial communities drawn ‘on religious or caste identity. Community identity seems to be the governing force. It is not therefore surprising that the so-called peripheral groups continue to maintain their identities with reference to the social groups (caste, religion or sect) to which they belong while getting involved in the political processes despite the fact that their political goals remain more or less identical. By helping to articulate the political voice of the marginalised, democracy in India has led to 'a loosening of social structures’ and empowered the peripherals to be confident of their ability to improve the socio-economic conditions in which they are placed. This is a significant political process that had led to a silent revolution through a meaningful transfer of power from the upper caste elites to various subaltern groups within the democratic framework of public governance’ 104. According to the passage, what does "deepening of democracy” mean in the Western context? (a) Consolidation of group and class identities (b) Democratisation translated as greater involvement of people (c) Democratisation as greater involvement of ‘atomized’ individuals in the public sphere (d) None of the above 105. Greater democratisation in India has not necessarily led to (a) the dilution of caste and communal identities in the public sphere (b) irrelevance of community identity as a governing force in Indian politics (c) marginalisation of elite groups in society (d) relative unimportance of hereditary identities over class identities 106. What is the “silent revolution” that has occurred in the Indian democratic process? (a) Irrelevance of caste and class hierarchies in political processes (b) Loosening of social structures in voting behaviour and patterns (c) Social change through transfer of power from upper caste elites to subaltem groups (d) All of the above Directions (Q.107-Q.109): Read and analyze the passage and attempt the following questions. For fourteen and a half months | lived in my little cell or room in the Dehradun jail, and | began to feel as if | was almost a part of it.| was familiar with every bit of it,| knew every mark and dent on the whitewashed walls and on the uneven floor and the ceiling with its moth-eaten rafters. In the little yard outside I greeted little tufts of grass and odd bits of stone as old friends. | was not alone in my cell, for several colonies of wasp and hornets lived there and many lizards found a home behind the rafters, emerging in the evenings in search of prey. 107. Which of the following explains best the sentence in the passage "! was almost a part of it"? (a) | was not alone in the cell (b) | was familiar with every bit of the cell (c) | greeted little tufts of grass like old friends. (d) | felt quite at home in the cell 108. The passage attempts to describe (a) the general conditions of the country’s jails (b) the prisoner's capacity to notice the minute details of his surroundings (c) the prisoner's conscious efforts to overcome the loneliness. (d) the prisoner's ability to live happily with other creatures mock cLaTits 4 LEGALEDGE 109. The author of the passage seems to suggest that (2) itis possible to adjust oneself to uncongenial surroundings (b) the conditions in Indian prisons are not bad (0) itis not difficult to spend one's time in a prison (d) there is a need to improve the conditions in our Jails Directions (Q.110-Q.117): Read and analyze the arguments and attempt the following questions. 110. Studies show that repeated exposure to an allergen can trigger an allergic person's defense mechanisms to the extent that eventually, even minimal exposure to the allergen can provoke a stronger than normal, even life threatening reaction Which of the following is the best analogy for the process by which minimal contact with an allergen can ‘cause a major reaction? (a) People with unhealthy diets are more susceptible to diabetes and high blood pressure (b) Because minor earthquakes can weaken the existing infrastructure in an area, a series of minor earthquakes can sometimes lead to more damage than a single major earthquake. (c) Peanut allergy sufferers should avoid all potential contact with peanuts. (d) A student who fails a test can still earn a good grade if she works hard for the rest of the semester. 111, The typological theory of species classification, which has few adherents today, distinguishes species solely on the basis of observable physical characteristics, such as plumage color, adult size, or dental structure. However, there are many so-called “sibling species,” which are indistinguishable on the basis of their appearance but cannot interbreed and thus, according to the mainstream biological theory of species classification, are separate species, Since the typological theory does not count sibling species as separate species, it is unacceptable. The reasoning in the argument is most vulnerable to criticism on the grounds that (a) the argument does not evaluate all aspects of the typological theor (b) the argument confuses a necessary condition for species distinction with a sufficient condition for species distinction (©) the argument, in its attempt to refute one theory of species classification, presupposes the truth of an opposing theory (d) the argument takes a single fact that is incompatible with a theory as enough to show that theory to be false 112. Jane: Professor Harper's ideas for modifying the design of guitars are of no value because there is no general agreement among musicians as to what a guitar should sound like and, consequently, no widely accepted basis for evaluating the merits of a guitar's sound. Mark: What's more, Harper's ideas have had enough time to be adopted if they really resulted in superior sound. It took only ten years for the Torres design for guitars to be almost universally adopted because of the improvement it makes in tonal quality. Which one of the following most accurately describes the relationship between Jane's argument and Mark's argument? (a) Mark’s argument shows how a weakness in Jane's argument can be overcome. (b) Mark’s argument has a premise in common with Jane's argument. (c) Mark and Jane use similar techniques to argue for different conclusions. (d) Mark’s argument and Jane's argument are based on conflicting suppositions 113. In an experiment, two-year-old boys and their fathers made pie dough together using rolling pins and other utensils. Each father-son pair used a rolling pin that was distinctively different from those used by the other fatherson pairs, and each father repeated the phrase “rolling pin" each time his son used it, But when the children were asked to identify all of the rolling pins among a group of kitchen utensils that included several rolling pins, each child picked only the one that he had used. Which one of the following inferences is most supported by the information above? (a) The children did not grasp the function of a rolling pin. (b) No two children understood the name “rolling pin" to apply to the same object. (c) The children understood that all rolling pins have the same general shape, mock cLaTits 4 LEGALEDGE (d) Each child was able to identify correctly only the utensils that he had used. 114. Max: Although doing so would be very costly, humans already possess the technology to build colonies on the Moon. As the human population increases and the amount of unoccupied space available for constructing housing on Earth diminishes, there will be a growing economic incentive to construct such colonies to house some of the population. Thus, such colonies will almost certainly be built and severe overcrowding on Earth relieved. Max's argument is most vulnerable to criticism on which one of the following grounds? (a) It takes for granted that the economic incentive to construct colonies on the Moon will grow sufficiently to cause such a costly project to be undertaken. (b) It takes for granted that the only way of relieving severe overcrowding on Earth is the construction of colonies on the Moon (0) It overlooks the possibility that colonies will be built on the Moon regardless of any economic incentive to construct such colonies to house some of the population (d) It overlooks the possibility that colonies on the Moon might themselves quickly become overcrowded 415. The importance of the ozone layer to terrestrial animals is that it entirely filters out some wavelengths of light but lets others through. Holes in the ozone layer and the dangers associated with these holes are well documented. However, one danger that has not been given sufficient attention is that these holes could lead to severe eye damage for animals of many species. Which one of the following is most strongly supported by the statements above, if they are true? (@) All wavelengths of sunlight that can cause eye damage are filtered out by the ozone layer where it is intact, (b) Few species of animals live on a part of the earth's surface that is not threatened by holes in the ozone layer. (0) Some species of animals have eyes that will not suffer any damage when exposed to unfiltered sunlight. (d) Some wavelengths of sunlight that cause eye damage are more likely to reach the earth's surface where there are holes in the ozone layer than where there are not. 116. One of the most vexing problems in historiography is dating an event when the usual sources offer conflicting chronologies of the event. Historians should attempt to minimize the number of competing sources, perhaps by eliminating the less credible ones. Once this is achieved and several sources are left, as often happens, historians may try, though on occasion unsuccessfully, to determine independently of the usual sources which date is more likely to be right. Which one of the following inferences is most strongly supported by the information above? (a) We have no plausible chronology of most of the events for which attempts have been made by historians to determine the right date. (b) Some of the events for which there are conflicting chronologies and for which attempts have been made by historians to determine the right date cannot be dated reliably by historians. (c) Attaching a reliable date to any event requires determining which of several conflicting chronologies is most likely to be true. (d) Determining independently of the usual sources which of several conflicting chronologies is more likely to be right is an ineffective way of dating events. 117. The increasing complexity of scientific inquiry has led to a proliferation of multi-authored technical articles. Reports of clinical trials involving patients from several hospitals are usually coauthored by physicians from each participating hospital. Likewise, physics papers reporting results from experiments using subsystems developed at various laboratories generally have authors from each laboratory. Ifall of the statements above are true, which one of the following must be true? (@) Clinical trials involving patients from several hospitals are never conducted solely by physicians from just one hospital (b) Most reports of clinical trials involving patients from several hospitals have multiple authors, (c) When a technical article has multiple authors, they are usually from different institutions. (d) Physics papers authored by researchers from multiple laboratories usually report results from experiments using subsystems developed at each laboratory. mock cLaTits 4 LEGALEDGE Directions (Q.118 and Q.119): Read and analyze the argument and attempt the following questions. The central argument for deconstruction depends on relativism, by which I mean the view that truth itself is always relative to the differing standpoints and predisposing intellectual frameworks of the judging subject. Indeed, to attempt to define deconstruction is to defy another of its main principles - which is to deny that final or true definitions are possible, because even the most plausible candidates will always invite a further defining move, or'play1, with language. For the deconstructor, the relationship of language to reality is not given, or even reliable. 418. Which of the following, if true, lends greatest support to the opinion of the deconstructor? (a) All languages have some words that are similar and show little if any change. (b) All language systems are based on logically relevant theories of their time. (0) All language systems are inherently unreliable cultural constructs. (d) Language has essentially evolved due to the need to humans to communicate their deepest thoughts. 119. Which of the following if true weakens the opinion of the deconstructor? 1. The meaning of a word has its origin in the structure of reality itself 2. Words attain new meanings with time due to changes in their relevance. (a) Only 1 (b) Only 2 (0) Both 1 and2——(d) Neither 1 nor 2 Directions (Q.120 and Q.121): Read and analyze the passage and attempt the following questions. It may seem counterintuitive that in film, where there's so much more money at stake, producers are more inclined to invest in new properties, while TV viewers have to make do with dusty reruns and the likes of Merry Madagascar. It's ail about supply and demand. Over the long winter break, parents willing to take the kids to the multiplex will see anything with Santa in it, so there's no shortage of demand for something new, But when it comes to TV they are content with old TV shows. 120. Which of the following, if true, strengthens the argument? 1. Most parents feel more comfortable with the TV shows they remember from when they grew up. 2, Parents fear that the content of the new serials will produce a negative effect on the minds of their children, (a) Only 2 (b) Onlyt () Neither 1 nor 2. (d) Both 1 and 2 121. Which of the following is a valid assumption? (a) Old TV shows are as popular as new films. (b) Investing in TV shows is pragmatic, (c) Old TV shows are as good as new films. (d) None of these. Directions (Q.122-Q.128): Read and analyze the arguments and attempt the following questions. 122, Unless the building permit is obtained by February 1 of this year or some of the other activities necessary for construction of the new library can be completed in less time than originally planned, the new library will not be completed on schedule. It is now clear that the building permit cannot be obtained by February 1, 80 the new library will not be completed on schedule. The conclusion drawn follows logically from the premises if which one of the following is assumed? (@) All of the other activities necessary for construction of the library will take at least as much time as originally planned. (b) The officials in charge of construction of the new library have admitted that it probably will not be completed on schedule. (0) The application for a building permit was submitted on January 2 of this year, and processing building permits always takes at least two months. (d) The application for a building permit was rejected the first time it was submitted, and it had to be resubmitted with a revised building plan. mock cLaTits 4 LEGALEDGE 123. After being subjected to clinical tests like those used to evaluate the effectiveness of prescription drugs, a popular non-prescription herbal remedy was found to be as effective in treating painful joints as is a certain prescription drug that has been used successfully to treat this condition, The manufacturer of the herbal remedy cited the test results as proof that chemical agents are unnecessary for the successful treatment of painful joints. The test results would provide the proof that the manufacturer claims they do if which one of the following is assumed? (@) People are likely to switch from using prescription drugs to using herbal remedies if the herbal remedies are found to be as effective as the prescription drugs. (b) The herbal remedy contains no chemical agents that are effective in treating painful joints. (c) None of the people who participated in the test of the prescription drug had ever tried using an herbal remedy to treat painful joints. (d) The researchers who analyzed the results of the clinical testing of the herbal remedy had also analyzed the results of the clinical testing of the prescription drug. 124, Philosopher: We should not disapprove of the unearthing of truths that we would rather not acknowledge or that, by their dissemination, might influence society in pernicious ways Which one of the following conforms most closely to the principle stated by the philosopher? (@) A law enforcement officer should not act upon illegally obtained information, even though such action might, in some cases, result in a benefit to society, (b) Scientific research should not be restricted even if it could lead to harmful applications, such as the manufacture of sophisticated weapons. (0) A physician should never withhold the truth from a patient, except in cases where depression induced by bad news might significantly affect the patient's recuperation. (d) Investigative journalists who employ illegal means of obtaining information should not be subjected to moral disapproval, if the revelation of that information does more good for society than does its suppression 125, Environmentalist: The excessive atmospheric buildup of carbon dioxide, which threatens the welfare of ‘everyone in the world, can be stopped only by reducing the burning of fossil fuels. Any country imposing the strict emission standards on the industrial burning of such fuels that this reduction requires, however, would thereby reduce its gross national product. No nation will be willing to bear singlehandedly the costs of an action that will benefit everyone. It is obvious, then, that the catastrophic consequences of excessive atmospheric carbon dioxide are unavoidable unless____ Which one of the following most logically completes the argument? (2) all nations become less concerned with pollution than with the economic burdens of preventing it (b) muttinational corporations agree to voluntary strict emission standards (c) intemational agreements produce industrial emission standards (d) distrust among nations is eliminated 126. Landscape architect: If he screen between these two areas is to be a hedge, that hedge must be of either hemlocks or Leyland cypress trees. However, Leyland cypress trees cannot be grown this far north. So if the screen is to be a hedge, it will be a hemlock hedge. In which one of the following is the pattem of reasoning most similar to that in the landscape architect's argument? (a) If there is to be an entrance on the north side of the building, it will have to be approached by a ramp. However, a ramp would become impossibly slippery in winter, so there will be no entrance on the north side. (b) If visitors are to travel to this part of the site by automobile, there will be a need for parking spaces. However, no parking spaces are allowed for in the design. So if visitors are likely to come by automobile, the design will be changed (0) The subsoil in these five acres either consists entirely of clay or consists entirely of shale, Therefore, if one test hole in the area reveals shale, it will be clear that the entire five acres has a shale subsoil (d) Any path along this embankment must be either concrete or stone. But a concrete path cannot be built in this location. So if there is to be a path on the embankment, it will be a stone path mock cLaTits 4 LEGALEDGE 127. The human body produces energy by metabolizing oxygen. This metabolic process forms free radicals as a natural by-product. Free radicals are individual atoms or sometimes groups of atoms that have unbalanced or unpaired electrons. These free radicals are volatile particles and they rob electrons from other molecules and cells within the body, causing cell damage that manifests itself as disease and aging. Which of the following, if true, weakens the argument about the baneful effects of free radicals most effectively? (a) Within our cells, we have a highly effective system that uses antioxidant compounds to control and counteract free radical damage. (b) There is not just one free radical, but a dizzying array of them at work in living organisms. (c) The wrecking ball qualities of free radical are often needed by cells and tissues to remove what is no longer useful and make way for the new. (d) None of the above 128. A study suggests that watching just nine minutes of a cartoon program on TV can cause short-term attention and leaming problems in 4-year-olds. In an experiment, 60 children were randomly assigned to either watch the cartoon show or to draw pictures. Immediately after these nine-minute assignments, the kids took mental function tests; those who had watched the cartoon did measurably worse than the others. The argument above depends on which of the following assumptions? (a) The children understood the assignments they were given. (b) Attention and other mental functions can be measured by tests. (c) A four year old can continuously watch a TV show for nine minutes without being distracted, (a) Very young children should be discouraged from watching too much television. 129. Film Critic: Movies have been remade for almost as long as they've been made. | wouldn't call it a curse The remake can bring the movie to a new younger audience and at times even improve upon the original film. Which of the following, if true, supports the claims made above? (a) Most remakes generate a lot of excitement and go on to do well financially. (b) Most remakes retain the distinctiveness of the original films and present it in a way relevant to the new generation. (c) Most remakes are very popular among fans of the original film. (d) Most remakes are negatively received by critics and are unsuccessful at the box office. Directions (Q.130-Q.133) Refer to the following statements and answer the questions Seven students Priya, Ankit, Raman, Sunil, Tony, Deepak and Vicky take a series of tests. No two; student get similar marks. Vicky always scores more than Priya. Priya always scores more than Ankit, Each time either Raman scores the highest and Tony gets the least or altematively Sunil scores the highest and Deepak or Ankit scores the least. 130. If Sunil is ranked sixth arid Ankit it ranked, fifth, which of the following can he true? (a) Vicky is ranked first or fourth (b) Raman is ranked second or third (0) Tony is ranked fourth or fifth (d) Deepak is ranked third or fourth, 131. If Raman gets the highest, Vicky should be ranked not lower than (@) Second (b) third (0) fourth (a) fifth 132. If Raman is ranked second and Ankit is ranked first, which of the following must be true (a) Sunil is ranked third (b) Tony is ranked third (0) Priya is ranked sixth (d) None of these 133. If Sunil is ranked second, which of the following can be true? LEGALEDGE MOCK CLAT #19 (a) Deepak gets more than Vicky (b) Vicky gets more than Sunil (0) Priya gets more than Raman (d) Priya gets more than Vicky Directions (2.134 and Q.135): Question consists of five statements followed by five conclusions. Consider the given statements to be true even if they seem to be at variance with commonly known facts. Read all the conclusions and then decide which of the given conclusions does not logically follow from the given statements using all statements together 134. Statements: All book are copy. Some copy are kite. No kite are hour. Some hour are time. All time are month, Conclusions: (a) Some month are hour. (b) All book being kite is a possibility (0) Some copy is not hour. (d) Some book is not hour. 135. Statements: Some fly are high. No high is world. All world is large. Some large is Note. All note is pen. Conclusions: (a) Allhigh being large is a possibility. (b) All fly being world is a possibility. (0) Some penis large. (d) All world being pen is a possibilty. 136. 437. 138. 139, 140. mock cLaTits 4 LEGALEDGE SST Directions (Q.136-@.140): Study the following information carefully and answer the given questions, ‘Among 400 cricket players, 45% played in IPL 1 and 6.25% played only in IPL 1. Again, 57.5% players played in IPL 2 and 11.25% players played only in IPL 2. Again, 72.5% players played in IPL 3 and 27.5% players played only in IPL 3. Twenty per cent players played in all three IPL tournaments. How many players are there who played in only IPL 1 and IPL 2 but not in IPL 3? (a) 30 (b) 35 (c) 40 (a) 65 What is the percentage of players who played in only IPL 2 and IPL 3 but not in IPL 1? (a) 12.5% (b) 16.25% (0) 22.75% (d) 24% What is the percentage of the players who played in at least two IPL tournaments? (a) 30% (b) 35% (0) 45% (4) 55% The number of players who played only in either IPL 1 or IPL 2 is what percentage of the players who played in all three IPL? (@) 72% (b) 75% (6) 87.5% (a)85% The number of players who played in at most one IPL tournament is what percentage more/less than the number of players who played in at least one IPL? (a) 55% more (b) 55% less (c) 40% more (d) 40% less WEST mock cLaTits 4 LEGALEDGE Directions (Q.141- 145): Following Venn-diagram shows the result of a survey conducted on people about their interest in different sports, Answer the following questions based on this diagram. Hockey Soccer Tennis ‘Total number of people = 500 141. How many people are there who like exactly three types of sports out of the given four? (a98 (b)104 (08 (112 142. The number of people who like exactly two types of sports is what percentage of the total number of people surveyed? (a)40% (b)32% (0)30% (4)26% 143. What's the difference between the number of people who like either only Cricket or only Soccer and the number of people who like either only Hockey or onlyTennis? (a1 (b)3 (95 (a7 144, What is the ratio of the number of people who like Cricket to the number of people who like only Tennis? (a) 40:13 (b) 59:16 (0) 63:23 (d) 64:25 145, The number of people who like exactly one type of sports is what percentage of the total number of people surveyed? (a}42.5% (b)45.4% (047.8% (451.1% u mock cLaTits 4 LEGALEDGE Directions (Q.146-Q.150): Following table shows the total number of students appeared from different cities, ratio of boys and girls among those appeared students, percentage of passed students and number of passed girls among them: [Total ‘Apeared | Pass% [Number of lAppeared | Boys : Girls girls passed $4|7210 3:2 60% 7268 'S2)4800 a7 66% 146 'S3{5670 5:4 70% 1432 S4{6400 15 68% 975 '$5{7200 11:7 57% 1224 ‘S@l7080 75 5% 7565 146. What is the average number of boys appeared in the examination from all six cities? (a) 3851 (b) 3852 (c) 3853 (d) 3854 147. The total number of girls passed from City Sq is what percentage of the total number of girls appeared from City Sq? (@) 43.25% (b) 48.75% (0) 52.5% (4) 55% 148. What is the total number of boys failed in the examination from alll six cities together? (a) 6175, (b) 6180 (c) 6185 (a) 6195 149, Thetotalnumberofgirispassedintheexaminationisapproximatelywhatpercentageofthe total number of girls appeared in the examination, taking all cities together? (a) 42% (b) 50% (0) 56% (4) 64% 150. The total number of boys passed from City S2 is what percentage more than the total number of girls passed from that city? (@) 70.2% (b) 76.5% (a) 80% u mock cLaTits 4 LEGALEDGE ROUGH WORK mock cLaTits 4 LEGALEDGE ROUGH WORK

You might also like